if a train travels 420 miles in 24 hours, what is its unit rate of travel

Answers

Answer 1

Answer:

The unit rate of travel is 17.5 miles per hour.

Step-by-step explanation:

Hope this helps!

Answer 2

The unit rate of train that travels 420 miles in 24 hours is 17.5 miles/hr.

What is speed?

Speed is defined as the rate of change of distance covered with respect to time. Mathematically -

Speed [S] = Distance [D] / time [T]

Given is a train that travels 420 miles in 24 hours.

Unit rate means how many miles did the train covered in 1 hour.

From the definition of speed, we can write -

Speed = Distance/time

Mathematically -

[S] = [D] / [T]

[S] = 420/24

[S] = 17.5 miles/hr

Therefore, the unit rate of train that travels 420 miles in 24 hours is 17.5 miles/hr.

To solve more questions on Kinematics, visit the link below-

https://brainly.com/question/96118

#SPJ5


Related Questions

What are the tax consequences to Euclid from the following independent events? In your computations, do not round intermediate division. If required, round the per share answer to two decimal places. Round all other answers to the nearest dollar. a. Euclid bought 500 shares of common stock five years ago for $50,000. This year, Euclid receives 20 shares of common stock as a nontaxable stock dividend. As a result of the stock dividend, Euclid's per share basis is $ X. b. Assume instead that Euclid received a nontaxable preferred stock dividend of 20 shares. The preferred stock has a fair market value of $5,000, and the common stock, on which the preferred is distributed, has a fair market value of $75,000. After the receipt of the stock dividend, the basis of the preferred stock is $ X, and the basis of the common stock is Φ

Answers

Euclid receives 20 shares of common stock as a nontaxable stock dividend.The basis of the common stock remains the same as in scenario a, which is $96.15 per share.

To calculate the per share basis, we divide the original purchase cost by the total number of shares (including the dividend shares).  In scenario b, Euclid receives a nontaxable preferred stock dividend of 20 shares. The preferred stock has a fair market value of $5,000, and the common stock, on which the preferred is distributed, has a fair market value of $75,000.

The tax consequences involve determining the new basis of the preferred stock and the common stock after the dividend. a. To find the per share basis of Euclid's common stock after receiving the stock dividend, we divide the original purchase cost by the total number of shares. The original purchase cost was $50,000 for 500 shares, which means the per share basis was $50,000/500 = $100. After receiving 20 additional shares as a dividend, the total number of shares becomes 500 + 20 = 520.

Therefore, the new per share basis is $50,000/520 = $96.15. b. In this scenario, Euclid receives a preferred stock dividend of 20 shares. The preferred stock has a fair market value of $5,000, and the common stock has a fair market value of $75,000. To determine the new basis of the preferred stock, we consider its fair market value.

Since the preferred stock dividend is nontaxable, its basis is equal to the fair market value, which is $5,000.

Learn more about shares here: brainly.com/question/32277750

#SPJ11

The content dimension of a message deals with the _____ of a message, while the relational dimension of a message deals with the _____ of a message.

Answers

The content dimension of a message deals with the what of a message, while the relational dimension of a message deals with the how of a message is the correct answer.

Content dimensions is a generic concept to have multiple variants of a content node. It involves the information being explicitly discussed. The content repository supports any number of dimensions. The content dimension is the meaning of the actual message itself.

"Relational dimension of communication is a concept in which we have similar weight and impact as that of the message. This concept deals with the quality or nature of the relationships and networks."

Hence we can conclude that the content dimension of a message deals with the what of a message, while the relational dimension of a message deals with the how of a message is the correct answer.

Learn more about content dimension of a message here

https://brainly.com/question/10147211

#SPJ4

Two customers spent the same total amount of money at a restaurant.
• The first customer bought 8 hot wings and left a $4 tip.
• The second customer bought 10 hot wings and left a $2.80 tip.
• Both customers paid the same amount per hot wing.
How much does one hot wing cost at this restaurant in dollars and cents?
Record your answer in the boxes below. Be sure to use the correct place value.
+/-

Answers

Answer: $0.60 per wing

Explanation:
We’ll solve this with a set of equations since the price per wing is fixed and they paid the sam amount at the restaurant. Let x represent the cost of 1 wing.

8x + 4.00 = 10x + 2.80
4.00 = 2x + 2.80
1.20 = 2x
0.60 = x

So, we have found that one wing cost $0.60.

Cheers.

what are the area and perimeter of 7in 16in​

Answers

The Area and Perimeter of the rectangle is 112 Inches² and 46 Inches respectively.

Suppose in question they are talking about rectangle.

So, let

Length (l) of rectangle  = 16 Inches

Width (w) of rectangle  =   7 Inches

As we have both sides of rectangle now we will determine the value of area and perimeter of rectangle.

Area of rectangle = l × w

                              = 16 × 7

                              = 112 Inches²

Perimeter of rectangle = 2( l + w )

                                      = 2( 16 + 7 )

                                      = 2 × 23

                                      = 46 Inches

To learn more about Area and Perimeter visit : https://brainly.com/question/11957642

#SPJ9

a. what is the area of the window seat when x=3
b. write a polynomial that helps represent the area of the window seats

a. what is the area of the window seat when x=3b. write a polynomial that helps represent the area of

Answers

a) The area of the window seat when x=3 is 8 ft².

b) Polynomial that represent area of the window is  x² - 1

What is trapezium?

In American and Canadian English, a quadrilateral with at least one pair of parallel sides is referred to as a trapezoid. It is referred to as a trapezium in British and other varieties of English. In Euclidean geometry, a trapezoid is a convex quadrilateral by definition. The bases of the trapezoid are the parallel sides.

Area of trapezium = 1/2 (a+b)*h

a= x ft

b = x+2 ft

c = x-1 ft

a ) When x=3;

a = 3 ft

b = 3 + 2 = 5 ft

c = 3-1 = 2 ft

Now, Area of window seat = 1/2 (3+5) 2

= 1/2 * 8 * 2

= 8 ft²

Hence, area of window seat is 8 ft².

b) Polynomial that helps represent the area of window seats

= 1/2(a+b)*h

= 1/2(x + x +2) * (x-1)

= 1/2 (2x+2)*(x-1)

= 1/2 * 2(x+1)* (x-1)

= (x+1) * (x-1)

= x² - 1

Hence, the Polynomial that represent area of the window is  x² - 1.

To know more about polynomial check the below link:

https://brainly.com/question/4142886

#SPJ1

Deltamath

Please help me

Deltamath Please help me

Answers

5,-1 I believe is the correct answer

Answer:

It should be (6, -3) If I am correct- sorry if it isn't

Solve for x.
Use the completing the square method.
x2 + 10x = 6

Answers

x2+10x=6

x2+10x-6=0

x=-10+-Square root of [10^2-4×(-6)]/2.1

x=-10+-Square root of 124/2

x=-5+square root of 31 and -5-square root of 31

Write 95cents as a percentage of $2.25.

Answers

Answer:

Therefore, 95 cents is 42.22% of $2.25.

Step-by-step explanation:

To find the percentage of 95 cents in $2.25, we can use the following formula:

Percentage = (part / whole) x 100%

Here, the part is 95 cents and the whole is $2.25. We first need to convert 95 cents to dollars by dividing it by 100:

95 cents / 100 = $0.95

Now we can plug in the values into the formula and solve for the percentage:

Percentage = ($0.95 / $2.25) x 100%

Percentage = 42.22%

dave is 15 years old and his uncle rob is three times as old. when dave will be 32 years old, his uncle rob will be

Answers

By considering ages and equations we have,

Uncle Rob will be 62 years old when Dave is 32.

Dave is listed as being 15 years old, while his uncle Rob is listed as being three times Dave's age. Rob's age as of right now may be calculated using the formula below:

Age of Rob is Dave times three.

Using Dave's age as a plug-in, we obtain following equations,

Rob's age is equal to 15 + 3 = 45.

Rob is currently 45 years old, according to this.

We need to know how many years it will be before Dave turns 32 in order to calculate Rob's age at that time. By deducting Dave's present age from his anticipated age, we may determine this:

Years from now = Dave's age in the future minus Dave's age currently

By entering the specified values, we obtain:

The number of years from now is equal to 32 - 15 = 17 years.

Dave will turn 32 in 17 years, according to this.

We may multiply Rob's current age by the number of years from now to determine his age at that point:

When Dave reaches the age of 32, Rob will be that age plus the number of years.

When we enter the calculated values, we obtain:

When Dave is 32 years old, Rob will be 45 + 17 years old, or 62 years old.

  So, Dave uncle rob 's age will be 62 years old.

As a result, when Dave's age is 32, his uncle Rob's age will be 62.

To know more about equations at :

brainly.com/question/27849342

#SPJ4

The area of the shaded circle below is 78.5 square inches. The area of the large circle is 314 square inches. A shaded circle is inside of a larger unshaded circle. What is the probability that a point chosen at random will be in the shaded region? Round the answer to two decimal places. 0.25 0.33 0.66 0.75

Answers

The probability that a point is chosen at random in the shaded region will be 0.25.

What is probability?

Its simple notion is that something will most likely occur. The favorable event's proportion to the overall number of occurrences.

Probability = (Favorable event) / (Total event)

The area of the shaded circle below is 78.5 square inches.

The area of the large circle is 314 square inches.

A shaded circle is inside of a larger unshaded circle.

Then the probability that a point is chosen at random in the shaded region will be

P = 78.5 / 314

P = 1 / 4

P = 0.25

Then the correct option is A.

More about the probability link is given below.

https://brainly.com/question/795909

#SPJ1

Drag the tiles to the correct boxes to complete the pairs. Match each point of intersection with the system of equations whose solution is at that point.

Drag the tiles to the correct boxes to complete the pairs. Match each point of intersection with the

Answers

To determine which point is a solution to which equation system, you have to identfy which line corresponds to which equation and the coordinates of each point.

Points

W (≈-7/3,≈4/3)

X (-1,3)

Y (1,1)

Z (≈-1/2,-2)

The symbol ≈ indicates that is an approximate value.

Brown line

Has y-intercept at (0,-3)

And slope

Using points (0,-3) and (-1,-1)

\(m=\frac{-3--1}{0--1}=-\frac{2}{1}=-2\)

Its equation is y=-2x-3

Blue line

Has y-intercept (0,2)

And slope

Using points (0,2) and (-1,-3)

\(m=\frac{2-3}{0--1}=-1\)

Its equation is y=-x+2

Red line

Has y-intercept (0,4)

And slope

Using points (0,4) and (-1,-3)

\(m=\frac{4-3}{0--1}=1\)

Its equation is y=x+4

Black line

Has y-intercept (0,-1)

And slope

Using points (0,-1) and (1,1)

\(m=\frac{1-(-1)}{1-0}=2\)

The equation for this line is y=2x-1

The first equation system is

y=-2x-3

y=2x-1

Corresponds to the intersection between the brown and black lines. The point that is a solution for this system is Z

The second equation system is

y=x+4

y=-x+2

Corresponds to the intersection between the blue and red lines. The point that is a solution for this system is X

The third equation system is

y=-2x-3

y=x+4

Corresponds to the intersection between the brown and red lines. The point that is a solution for this system is W

The fourth equation system is

y=2x-1

y=-x+2

Corresponds to the intersection between the black and blue lines. The point that is a solution for this system is Y

Another way for solving this exercise is by calculating the solution of each system and placing the results in the grid. For example for the first system:

y=-2x-3

y=2x-1

\(-2x-3=2x-1\)

solve for x

\(\begin{gathered} -2x-2x-3=2x-2x-1 \\ -4x-3+3=-1+3 \\ -4x=2 \\ -\frac{4x}{-4}=\frac{2}{-4} \\ x=-\frac{1}{2} \end{gathered}\)

And now replace this value in one of the equations

\(\begin{gathered} y=-2(-\frac{1}{2})-3 \\ y=-2 \end{gathered}\)

The solution for this system is (-1/2,-2) → If you look at the coordinates determined above, you'll see that these correspond to point Z

Both ways are equally valid to determine which point corresponds to each system.

Can someone please please help me I don’t understand

Can someone please please help me I dont understand

Answers

Answer: Pretty sure the answer is C

Step-by-step explanation:

How long is this triangles hypotenuse?

How long is this triangles hypotenuse?

Answers

Answer:

17.

Step-by-step explanation:

\(a^2+b^2 = c^2\) is the formula used to find the hypotenuse (also known as the Pythagora's Theorem).

Substitute the given values into the equation.

\(8^2+15^2=c^2\)

64 + 225 = \(c^2\)

289 = \(c^2\)

\(\sqrt{289}\) = c

c = 17

What is the sum of all the angles that are labeled?

Image of three angles around a single vertex. One angle is fifty five degrees, one is sixty four degrees, and one is one hundred seventy five degrees.

Answers

The sum of all the angles that are labeled in the given vertex is 294°

In a single vertex, the three angles around the vertex are given as 55°, 64°, and 175°

We have to find the sum of all the angles that are labeled in the given single vertex.

What is a vertex?

A vertex is a point two or more lines meets.

It is the corner of a geometrical shape.

Example:

A square has 4 corners so it has 4 vertexes.

A cube has 8 corners so it has 8 vertexes.

We will add all the different angles in the single vertex as shown in the figure below.

Let,

Angle A = 55°

Angle B = 64°

Angle C = 175°

The sum of all the angles that are labeled is:

= Angle A + Angle B + Angle C

= 55° + 64° + 175°

= 294°

The sum of all the angles that are labeled in the given vertex is 294°

Learn more about the vertex here:

https://brainly.com/question/12563262

#SPJ1

What is the sum of all the angles that are labeled?Image of three angles around a single vertex. One

A polling company conducts an annual poll of adults about political opinions. The survey asked a random sample of 361
adults whether they think things in the country are going in the right direction or in the wrong direction. 47​% said that things were going in the wrong direction.

a) Are the assumptions and conditions required to apply a confidence interval met? Select all that apply.
A. Yes, all assumptions and conditions are met.
B. No, because the sample is a simple random sample.
C. No, because there are less than 10 expected "successes" and 10 expected "failures."
D. No, because the sample is greater than 10% of the population.
E. No, because the sample is less than 10% of the population.
F. No, because there are at least 10 expected "successes" and 10 expected "failures."
G. No, because the sample is not a simple random sample.

Answers

In this case, option (F) is the correct choice, because there are at least 10 expected "successes" and 10 expected "failures" in the sample, which satisfies the requirement for constructing a confidence interval for a proportion.

The question provides information about a survey conducted by a polling company to measure political opinions.

The survey asked a random sample of 361 adults whether they think things in the country are going in the right direction or the wrong direction, and 47% responded that things were going in the wrong direction.

The question is asking whether the assumptions and conditions required to apply a confidence interval are met.

To apply a confidence interval, we assume that the sample is a simple random sample from the population of interest, and that the sample size is sufficiently large.

Moreover, for constructing a confidence interval for a proportion, we also require that there are at least 10 expected "successes" and 10 expected "failures" in the sample.

Option (B) is incorrect because a simple random sample is one of the assumptions required to apply a confidence interval, and the question states that the sample is a random sample.

Option (C) is incorrect because the sample size is large enough for constructing a confidence interval.

Option (D) and option (E) are incorrect because they do not accurately reflect the conditions required to apply a confidence interval for a proportion.

Option (A) and option (G) are not correct choices because they do not accurately address the assumptions and conditions required to apply a confidence interval for a proportion.

Therefore, the correct answer is (F), i.e., the assumptions and conditions required to apply a confidence interval are met, including the requirement of having at least 10 expected "successes" and 10 expected "failures".

For similar question on confidence interval.

https://brainly.com/question/30536583

#SPJ11

1

1 -

2

3

4

5

6

7

00

8

9

Suppose that the functions f and g are defined for all real numbers x as follows.

f(x) = 4x²

g(x) = x + 1

Write the expressions for (-8)(x) and (f g)(x) and evaluate (r+g)(-2).

.

(6-8)(x) =

(6.8) (x) = 1

(6+8) (-2) = 0

Х

5

?

Answers

If we add the outputs of f(-2) and g(-2), we get 27.2 - 1/7, which simplifies to 190.6/7 or approximately 27.23.

How to find?

So we have two functions here, f(x) and g(x), and they're defined for all real numbers x. The function f(x) is defined as 4x² times (6.8) - so we're basically multiplying 4x² by a constant value of 6.8. The function g(x) is defined as 1 divided by (6+8) times -2, which simplifies to -1/7.

When we plug in x=-2 into g(x), we get -1/7. For f(x), we're multiplying -2 by itself, which gives us 4, and then multiplying that by 6.8, giving us 27.2.

So, if we add the outputs of f(-2) and g(-2), we get 27.2 - 1/7, which simplifies to 190.6/7 or approximately 27.23.

To know more on Real number visit:

https://brainly.com/question/551408

#SPJ11

What is the y-intercept of f(x) = see
Picture

What is the y-intercept of f(x) = seePicture

Answers

Answer:

(0,1)

Step-by-step explanation:

f(x) = (1/2)^x

To find the y intercept, let x = 0

f(x) = (1/2) ^0

f(x) = 1

(0,1)

Which graph represents this system?
2x-5--5
y-3x+1

Which graph represents this system?2x-5--5y-3x+1

Answers

can't see the graphs....

please edit the picture if u can.

two cards are chosen at random from a deck of 52. what is the probability that both cards are numbers (2 through 10) totaling to 12

Answers

The probability is 0.3845.The probability of anything occurring is known as probability.

How are probabilities calculated?The probability of anything occurring is known as probability. To determine probability, divide the total number of possible outcomes by the number of possible ways an event could occur.The likelihood or chance that a specific event will occur is represented by a probability. Both proportions between 0 and 1 and percentages between 0% and 100% can be used to describe probabilities.To determine probability, divide the total number of possible outcomes by the number of possible ways an event could occur.

Explanation:

2/52 * 10/52  = 20/52 = 0.3845.

To learn more about probability refer to:

https://brainly.com/question/24756209

#SPJ4

If we change a 95% confidence interval estimate to a 99% confidence interval estimate, we can expect: A. the size of the confidence interval to decrease. B. the sample size to increase. C. the size of the confidence interval to increase. D. the size of the confidence interval to remain the same.

Answers

If we change a 95% confidence interval estimate to a 99% confidence interval estimate, we can expect the size of the confidence interval to increase.

This is because a higher level of confidence requires a wider interval to encompass a larger range of possible values. The sample size does not necessarily need to change to adjust the confidence interval. Therefore, the correct answer is C. the size of the confidence interval to increase.
If we change a 95% confidence interval estimate to a 99% confidence interval estimate, we can expect C. the size of the confidence interval to increase. This is because a higher confidence level requires a larger range to ensure the true population parameter is captured with more certainty.

To learn more about range visit;

brainly.com/question/29452843

#SPJ11

Point (w, z) is transformed by the rule (w 5, z) . what type of transformation occurred?

Answers

Scaling transformation onto the x co ordinate by a factor of 5.

What is a co-ordinate system ?

A co-ordinate system represents two points one for x axis and another for y axis in a ordered pair (x,y)

According the given question Point (w, z) is transformed by the rule (w5, z).

Here w represents x co-ordinate points and z represents y co-ordinate points.

We can observe that the x co-ordinate point have been scaled by a factor of 5.We can think of that after this transformation the line has become less steeper from origin.

learn more about transformation here :

https://brainly.com/question/11709244

#SPJ4

consider eigenstates of the momentum operator. the system is prepared in the stae
ψ=1/√6(φ2p+ φp)+√(2/3) φ-p
1. What are the possible result of a measurement of the kinetic energy K, and what are their respective probabilities?
2. Calculate the expectation value and the standard deviation of the kinetic energy.
3. What is the vector state after a measurement of the kinetic energy that has yielded the value kp=p^2/2M?

Answers

1. The possible results of a measurement of the kinetic energy given by the eigenvalues E(k) = k²/2m, and their respective probabilities are given by the expression above.

2. The expectation value and standard deviation of the kinetic energy are (5/12) (ℏ²/2m) and (1/2) (√35)/12) (ℏ²/m) respectively.

3. The vector state after the measurement is: |φkp⟩ = √(2πℏ) φp(x).

How to find possible results and respective probabilities?

1. To solve this problem, we first need to express the kinetic energy operator in terms of the momentum operator. In one dimension, the kinetic energy operator is given by:

K = p²/2m

where p is the momentum operator and m is the mass of the particle. We can then use the momentum eigenstates to express the state ψ in terms of the eigenstates of the kinetic energy operator.

To find the possible results of a measurement of the kinetic energy K and their respective probabilities, we need to express the state ψ in terms of the eigenstates of the kinetic energy operator. We can do this using the following relation:

K |k⟩ = E(k) |k⟩

where E(k) is the eigenvalue corresponding to the eigenstate |k⟩. The momentum eigenstates are also eigenstates of the kinetic energy operator, with eigenvalues E(k) = k²/2m. Therefore, we can express the state ψ in terms of the momentum eigenstates as:

ψ = 1/√6 (φ₂p + φp) + √(2/3) φ-p₁

= 1/√6 (|p₂⟩ + |p⟩) + √(2/3) |-p₁⟩

To find the possible results of a measurement of K, we need to project ψ onto the eigenstates of K and find the corresponding probabilities. The projection of ψ onto the eigenstate |k⟩ is given by:

⟨k|ψ⟩ = 1/√6 ⟨k|p⟩ + 1/√6 ⟨k|p₂⟩ + √(2/3) ⟨k|-p₁⟩

The probabilities of measuring the kinetic energy E(k) are then given by the squared magnitudes of the projections:

P(E(k)) = |⟨k|ψ⟩|²

We can simplify these expressions using the momentum eigenstates:

⟨k|p⟩ = √(ℏ/2π) \(e^(^-^i^k^x^)\)

⟨k|p2⟩ = (√(ℏ/2π))² (k²)\(e^(^-^i^k^x^)\)

⟨-k|p1⟩ = √(ℏ/2π) \(e^(^i^k^x^)\)

Substituting these expressions into the projection formula, we get:

⟨k|ψ⟩ = 1/√6 (√(ℏ/2π))² (k² + k) \(e^(^-^i^k^x^)\) + 1/√6 √(ℏ/2π) \(e^(^-^i^k^x^)\)+ √(2/3) √(ℏ/2π) \(e^(^i^k^x^)\)

Simplifying this expression, we get:

⟨k|ψ⟩ = (√(ℏ/2π)/√6) \([k^2 + k + \sqrt6 + 2\sqrt2 e^(^i^k^x^)]\)

The probability of measuring the kinetic energy E(k) is then given by:

P(E(k)) = |⟨k|ψ⟩|²

\(= (h/2\pi ) / 6 [k^4 + k^2 + 6k^2 + 2k^3\sqrt6 + 2k\sqrt6(k^2+1) + 8(k^2+1)]\)

Therefore, the possible results of a measurement of the kinetic energy K are given by the eigenvalues E(k) = k²/2m, and their respective probabilities are given by the expression above.

How to find expectation value and the standard deviation?

2. The expectation value of the kinetic energy is given by the formula:

⟨K⟩ = ⟨ψ|K|ψ⟩

Substituting the expression for ψ and K, we get:

⟨K⟩ = ⟨ψ|(p²/2m)|ψ⟩

= 1/6 ⟨φ₂p|p²|φ₂p⟩ + 1/3 ⟨φ₂p|p²|φp⟩ + 2/3 ⟨φ-p₁|p²|φ-p₁⟩

= 1/6 (ℏ/2)² (2/3)² + 1/3 (ℏ/2)² + 2/3 (ℏ/2)²

= (5/12) (ℏ²/2m)

To find the standard deviation of the kinetic energy, we first need to find the variance:

Var(K) = ⟨K²⟩ - ⟨K⟩²

We can find ⟨K²⟩ using the same formula as before:

⟨K²⟩ = ⟨ψ|(p²/2m)²|ψ⟩

Substituting the expression for ψ and K², we get:

⟨K²⟩ = 1/6 ⟨φ₂p|p⁴|φ₂p⟩ + 1/3 ⟨φ₂p|p⁴|φp⟩ + 2/3 ⟨φ-p₁|p⁴|φ-p₁⟩

= 1/6 (ℏ/2)⁴ (2/3)² + 1/3 (ℏ/2)⁴ + 2/3 (ℏ/2)⁴

= (5/4) (ℏ⁴/16m²)

Substituting these values into the formula for the variance, we get:

Var(K) = (5/4) (ℏ⁴/16m²) - [(5/12) (ℏ²/2m)]²

= (35/144) (ℏ⁴/16m²)

Finally, the standard deviation of the kinetic energy is given by the square root of the variance:

σ(K) = √(Var(K))

= √[(35/144) (ℏ⁴/16m²)]

= (1/2) (√35)/12) (ℏ²/m)

How to find vector state?

3. After a measurement of the kinetic energy that has yielded the value kp = p²/2m, the system is in an eigenstate of the kinetic energy operator with eigenvalue kp. Therefore, the vector state after the measurement is:

|φkp⟩ = N φp(x)

where N is a normalization constant and φp(x) is the eigenstate of the momentum operator with eigenvalue p. To find N, we use the normalization condition:

⟨φkp|φkp⟩ = 1

Substituting the expression for |φkp⟩ and using the normalization condition for φp(x), we get:

N² ⟨φp|φp⟩ = 1

N = 1/√⟨φp|φp⟩

Substituting the expression for φp(x), we get:

N = √(2πℏ)

Therefore, the vector state after the measurement is:

|φkp⟩ = √(2πℏ) φp(x)

where φp(x) is the eigenstate of the momentum operator with eigenvalue p = √(2mkp)/ℏ.

Learn more about kinetic energy

brainly.com/question/15764612

#SPJ11

Find the distance between the points (6,5 √ 2) and (4,3 √ 2)

Answers

Answer:

2\(\sqrt{2}\)

Step-by-step explanation:

distance formula = square root of the difference of the y-values, squared, plus the difference of the x-values, squared

= \(\sqrt{4 + 4}\)

= \(\sqrt{4}\) \(\sqrt{2}\)

= 2 \(\sqrt{2}\)

choose the best definition of hypothesis in the context of statistical analysis.

Answers

In statistical analysis, a hypothesis refers to a tentative explanation or prediction that is based on limited evidence and is subject to further investigation and testing. It is a statement that can be either true or false, and is typically formulated in such a way that it can be tested using statistical methods.

The hypothesis is often used to guide the research process, to help identify potential patterns or relationships in the data, and to evaluate the significance of the results.

Overall, the hypothesis plays a critical role in statistical analysis, as it provides a framework for understanding and interpreting data, and helps to ensure that research findings are reliable and valid.

A hypothesis in the context of statistical analysis is a tentative explanation or prediction about the relationship between two or more variables, which is subject to testing and empirical evaluation using statistical methods.

It is a statement that can be either true or false, and it is usually formulated in terms of the expected direction and strength of the relationship between the variables of interest.

The hypothesis is typically derived from existing theory, prior research, or common sense, and it serves as a guide for the collection, analysis, and interpretation of data in a scientific study.

The process of testing a hypothesis involves setting up null and alternative hypotheses, selecting an appropriate statistical test, collecting and analyzing data, and drawing conclusions based on the results of the analysis.

For similar question on hypothesis.

https://brainly.com/question/12416923

#SPJ11


Nadine has a plastic container in the shape of a prism that is filled with beads. The container has a length of 6 inches, a width of 8 inches, and a height of
5 inches
The container has a crack, so she needs to move all of the beads to new containers that are also rectangular prisms. Determine which sets of containers she could use for
the beads.
Select Yes or No for each set of containers.

Nadine has a plastic container in the shape of a prism that is filled with beads. The container has a

Answers

Answer:

BTW C= containers and AC = area of all the containers all together.

Step-by-step explanation:

So first off you calculate the area of the container that had a crack so its

Width X Length X height    |   6 x 8 x 5 = 240

So then you look at the possible answers and then calculate the area of those   plastic containers. So then you start with the first one and it would be like this

First possible answer: 3 and 1/3  x 6 x 5 = 100 x 3c = 300ac

Second possible answer: 2 and 1/4 x 4 and 3/4 x 6 = 64.125 x 4c = 256.5ac

Third possible answer: 3.5 x 6.5 x 6 = 136.5 x 2c = 273ac

Fourth possible answer: 3 and 1/2 x 3 and 1/4 x 5 = 56.875 x 5c = 284.375ac

Hopefully this will help you find your answer!! I tried my best!! Please give brainly!!

-64+k^2=0 solve this using quadratic formula​

Answers

Answer:

x= +8 or -8

Step-by-step explanation:

x=  -b±\(\sqrt{b^2-4ac}\) /2a

x=−0±\(\sqrt{0^2 negative4(1)( negative 64)}\) /√2(1)

x=−0±0\(\sqrt{256}\)/√2

x=−0±256−−−√2

x=−0±162

x=16/2 x=−16/2

x=8

x=−8

Jan is constructing an inscribed circle in a triangle. What is the center of this circle called?

Answers

Answer:

incenter

Step-by-step explanation:

The center of the circle inscribed in a triangle is the incenter of the triangle, the point where the angle bisectors of the triangle meet.

Which of the following coordinates represents a solution to the equation 3x-2y=6?

3, -2
2, -3
0, -2
(0, -3)

Answers

Answer:

2,-3

Step-by-step explanation:

Suppose you were given the function F(x) = x4 - 2x³ + 3x² - 10x + 3 and
the factor (x - 2). What is the value of a?

Answers

The  value of x =is 7.5.

What are function?

Any subset of a Cartesian product is a relation.

For instance, a subset of is known as a "binary relation from A to B," and in specifically, a subset of is known as a "relation on A."

These ordered pairs (a,b) with first components from A and second components from B make up a binary relation from A to B.

A function is a relation that links every item in a set X to a single item in a different set Y. (possibly the same set).

A graph, which is the collection of all ordered pairs (x, f (x)), serves as the sole representation of a function.

As you can see from these definitions, every function is a relation from X to Y, but not vice versa .

F(x)=x^4-2x^3+3x^2 - ax+3

F(2) = 2^4-2(2)^3+3(2)^2 - a(2) +3

F(2) = 16 - 16 + 12 - 2a +3

F(2) = 15 - 2a

15 - 2a = 0

15 = 2a

a = 7.5

Hence, The  value of x =is 7.5.

Learn more about function here:

brainly.com/question/2253924

#SPJ1

Omar has a new credit card. He earns 300 reward points for every 100100100 dollars he spends.
If Omar earned 1800 reward points, how much money did he spend?

Answers

Answer:

Omar spent $600

Step-by-step explanation:

He gets 300 points for every $100 he spends and if he has 1800 points total then you divide 1800 by 300 to get the answer

Other Questions
What historical events led to the Rwandan genocide? the verticals of a figure are given draw the figure and its image after a dilation with the given scale factor identify the type of dilatation A(-2,2) B(1,2) C(1,-1) Let X be a discrete random variable with range SX={1,2,3,4}. Let g(X)=IA, where A={X>2}, and let h(X)=max(X2,0). Suppose that the PMF of X satisfies the following condition for k=1,2,3:PX(k+1)=PX(k)/2. Find E[h(X)g(X)]. by studying the orbit of a moon around its parent planet, astronomers can use ________ to find the parent planets ________. Do you think the farmers had problems with industrialization (factories, machines)? Why or Why not? Vermicelli Company plans to sell 300,000 units of finished product in July and anticipates a growth rate in sales of 5% per month. The desired monthly ending inventory in units of finished product is 80% of the next month's estimated sales. There are 250,000 finished units in inventory on June 30th. Vermicelli Company's production requirement in units of finished product for the three-month period ending September 30th is: (CMA adapted) A 2kg body is moving a long a horizontal circular path of radius 2m with constant speed of 6m/s . what is the magnitude of the force acting on the body ? f(x) = x^2- 2x- 3g(x) = 5x^2Find f (g(x)) - Suppose you use your entrepreneurial spirit and economics training to start your own business. In your first year of work, you are able to earn $58,000 in gross income. What is the total amount of social insurance taxes you owe the federal government Out of the five pacific states which one is the largest Find the measure of angle f given the information below. PLS EXPLAIN YOUR REASON !!!! What was clevelands position on tariffs, and what did he do to promote this position?. Evaluate the integral I x f''(x) dx when f(0) = 2, f(1) = 3, f'(0) = 2, f'(1) = 4. I = 2 I = 4 I = 1 I = 1 I = 0 I = 3 Convert the decimal 0.92 to a fraction.Question 4 options:8310083908399299 Using the line of best fit for the above data, what is the approximate difference between the actual value for x = 10 and the predicted value for x= 10?A. 1.60B. 1.65 C. 1.70D. 1.75 What would have happened if Romeo didn't kill Tybalt? HELP DUE IN AN HOUR WILL GIVE BRAINLIST!!!!!!!! Define substrate-level phosphorylation.LO #3 (Set 5) Why 'The constitution of Nepal 2072 is regarded as the best constitution ever made in our country ? Write any eight reasons.